Filled Julia set - check my solution if it is right.

  • Thread starter Thread starter foxofdesert
  • Start date Start date
  • Tags Tags
    Set
Click For Summary
The discussion focuses on finding the value of R such that |f(z)| > |z| for |z| > R, where f(z) = z^3 - 27z + 15. The user proposes that R = 5 based on the roots of the equation z^3 - 28z + 15 = 0, identifying 5 as the furthest point. However, there is uncertainty about this solution, as it is suggested that at z = 5, f(5) = 0, indicating that |f(z)| may not exceed |z| in that vicinity. The conversation emphasizes the need to determine where |f(z)| - |z| > 0 to confirm the correct value of R. The user seeks validation of their approach and understanding of the problem.
foxofdesert
Messages
21
Reaction score
0

Homework Statement


Suppose f(z) = z^3 -27z + 15
Find R such that |f(z)|>|z| whenever |z|>R.

Homework Equations





The Attempt at a Solution


Let f(z)=z, then I have z^3 -28z + 15 = 0
then,

z=5, (-5+√(17))/2, (-5-√(17))/2.

since 5 is the most further point, R=5.

check my solution if it is right. I did it, but I am not so sure.
 
Physics news on Phys.org
so at z=5, f(5) = 0 and at z=5+e, for some small e>0, f will still be pretty small, so I don't think you've nailed it

you need to find where
|f(z)|-|z|>0
 
Question: A clock's minute hand has length 4 and its hour hand has length 3. What is the distance between the tips at the moment when it is increasing most rapidly?(Putnam Exam Question) Answer: Making assumption that both the hands moves at constant angular velocities, the answer is ## \sqrt{7} .## But don't you think this assumption is somewhat doubtful and wrong?

Similar threads

  • · Replies 9 ·
Replies
9
Views
2K
  • · Replies 7 ·
Replies
7
Views
1K
Replies
3
Views
2K
  • · Replies 10 ·
Replies
10
Views
2K
  • · Replies 8 ·
Replies
8
Views
2K
Replies
4
Views
2K
  • · Replies 2 ·
Replies
2
Views
2K
  • · Replies 3 ·
Replies
3
Views
2K
Replies
2
Views
1K
  • · Replies 4 ·
Replies
4
Views
4K